3
$\begingroup$

Let $S_{g,b}$ an orientable surface with genus $g$ and $b$ boundary components and $S_g^b$ be an orientable surface with $b$ punctures.

Denote by $PMCG(S_g^b)$ and $PMCG(S_{g,b}) $ the pure mapping class groups, that is, the group of orientation preserving homeomorphisms of the surface fixing the punctures or the boundary pointwise modulo isotopy.

Is $$0 \to \mathbb Z^b \to PMCG(S_{g,b}) \to PMCG(S_g^b) \to 0 $$ a split short exact sequence ?

$\endgroup$

1 Answer 1

4
$\begingroup$

No, it is not split (except in a few degenerate cases like $(g,n) = (0,1)$ or $(g,n)=(0,2)$; let's assume that $g \geq 2$ for the moment just to be careful). It is a central extension, so if it was split then the abelianization of the pure mapping class group of $S_{g,b}$ would contain a copy of $\mathbb{Z}^b$; however, this abelianization is trivial.

$\endgroup$
4
  • $\begingroup$ Thank you for your answer. Could you clarify your point about abelianizations and precise why the abelianization of $PMCG(S_{g,b})$ is trivial? $\endgroup$
    – Anonymous
    Oct 15, 2013 at 13:18
  • $\begingroup$ I am sorry, but there a still a couple of things I still don't get. I'll write here, there may be naive mistakes, please correct me if I am wrong. $PMCG(S_g^b)$ is generated by Dehn twists around non-trivial simple closed curves, right? Every non-trivial simple closed of $S_g^b$ can be seen as a non-trivial simple closed curve in $S_{g,b}$, so it seems that $PMCG(S_g^b)$ is "naturally" a subgroup of $PMCG(S_{g,b})$. So why does the inclusion $PMCG(S_{g}^b) \to PMCG(S_{g,b})$ not split the sequence above? Thank you in advance. $\endgroup$
    – Anonymous
    Oct 15, 2013 at 13:19
  • 1
    $\begingroup$ @yanglee : For your first question, see any standard source on the mapping class group (eg Farb-Margalit). For your second question, try to write down a careful construction of your inclusion and you will see that it is not well-defined even though it is well-defined on Dehn twists. $\endgroup$ Oct 15, 2013 at 16:21
  • $\begingroup$ I see the problem now, thank you very much! $\endgroup$
    – Anonymous
    Oct 15, 2013 at 17:09

Your Answer

By clicking “Post Your Answer”, you agree to our terms of service and acknowledge that you have read and understand our privacy policy and code of conduct.

Not the answer you're looking for? Browse other questions tagged or ask your own question.